You are on page 1of 60

ACE

Engineering Academy
Hyderabad | Delhi | Bhopal | Pune | Bhubaneswar | Bengaluru | Lucknow | Patna | Chennai | Vijayawada | Visakhapatnam | Tirupati | Kukatpally | Kolkata

H.O: 204, II Floor, Rahman Plaza, Opp. Methodist School, Abids, Hyderabad‐500001,
Ph: 040-23234418, 040-23234419, 040-23234420, 040 - 24750437

ACE Pre-GATE 2017 Branch: EE


Q.1 – Q.5 Carry One Mark Each
01. Reaching a place of appointment on Friday. I found that I was two days earlier than the scheduled
day. If I had reached on the following Wednesday then how many days late would I have been?
(a) one (b) Two
(c) three (d) four
01. Ans: (c)
Sol: Friday  2 days earlier
Therefore, scheduled day = Friday + 2 = Sunday
Sunday + 3 = Wednesday
Therefore, I would have been late by 3 days

02. Choose the most appropriate phrase from the options given below to complete the following
sentence.
The bus stopped to _________ more passengers.
(a) Take in (b) Take on
(c) Take up (d) Take for
02. Ans: (b)

03. Choose the appropriate sentence from the following options.


(a) She has been discharged since.
(b) She has since been discharged.
(c) She has been since discharged.
(d) She since has been discharged.
03. Ans: (b)
ACE Engineering Academy Hyderabad|Delhi|Bhopal|Pune|Bhubaneswar| Lucknow|Patna|Bengaluru|Chennai|Vijayawada|Vizag |Tirupati | Kukatpally| Kolkata
:2: Pre GATE - 2017_EE

04. Fill in the blank with an appropriate phrase.


The jet _________ into the air.
(a) Soared. (b) Soured.
(c) Sourced. (d) Sored.
04. Ans: (a)

05. Choose the most appropriate word from the options given below to complete the following
sentence.
If I had known that you were coming, I ____________ you at the airport.
(a) Would meet (b) Would have met
(c) Will have met (d) Had met
05. Ans: (b)

ACE Engineering Academy Hyderabad|Delhi|Bhopal|Pune|Bhubaneswar| Lucknow|Patna|Bengaluru|Chennai|Vijayawada|Vizag |Tirupati | Kukatpally| Kolkata


:3: Pre GATE - 2017_EE

Q.6 – Q.10 Carry two marks each

06. Which of the following can be logically inferred from the given statement.
“No other studied medicine except Helen”
(a) Helen only studied medicine
(b) Only Helen studied medicine
(c) Helen studied only medicine
(d) Helen studied medicine only
06. Ans: (b)

07. The average electricity bill of a household for January to June is ₹ 980, for July to September is
₹ 670, for October to December is ₹ 720. If the family goes on vacation for June and July and no
electricity is used, what would be the average electricity bill for that year?
(a) ₹ 500 (b) ₹ 600 (c) ₹ 700 (d) ₹ 800

07. Ans: (c)


Sol: Average electricity bill from January to June = ₹ 980
 Total electricity bill from January to May = 980 × 5 = ₹ 4900
(As no electricity is used in June)
Similarly, total electricity bill from August to September (as no electricity is used in July)
= 670 × 2 = ₹ 1340
And total electricity bill from October to December = 720 × 3 = ₹ 2160
Therefore, total electricity bill from January to December = 4900 + 1340 + 2160 = ₹8400
8400
Thus, average electricity bill for the whole year   ₹700
12

08. The following question has four statements of three segments each. Choose the alternative where
the third segment in the statement can be deduced using both the preceding two but not just from
one of them.
A. Sonia is an actress. Some actresses are pretty. Sonia is pretty.
B. All actors are pretty. Manoj is not an actor. Manoj is not pretty

ACE Engineering Academy Hyderabad|Delhi|Bhopal|Pune|Bhubaneswar| Lucknow|Patna|Bengaluru|Chennai|Vijayawada|Vizag |Tirupati | Kukatpally| Kolkata


:4: Pre GATE - 2017_EE

C. Some men are cops. Some men are brave. Some brave people are cops.
D. All cops are brave. Some men are cops. Some men are brave.
(a) only C (b) only A (c) only D (d) B and C
08. Ans: (c)
Sol: Statements:
All cops are brave
Some men are cops

Men
Cops
Brave

Conclusion:
Some men are brave (True)
Hence, only D follows.

09. A contractor, who got the contract for building the flyover, failed to construct the flyover in the
specified time and was supposed to pay ₹ 50,000 for the first day of extra time. This amount
increased by ₹ 4,000 each day. If he completes the flyover after one month of stipulated time, he
suffers a loss of 10% in the business. What is the amount he received for making the flyover in
crores of rupee? (One month = 30 days)
(a) 3.1 (b)3.24 (c) 3.46 (d) 3.68

09. Ans: (b)


Sol: The sum of money that the contractor was supposed to pay for the period of an month over the
stipulated time is

n
 Sn  [2a  (n  1)d]
2

a = 50,000, n = 30, d = 4000


30
S30  [2  50,000  (30  1)  4000]  15[100,000  29  4000]
2
ACE Engineering Academy Hyderabad|Delhi|Bhopal|Pune|Bhubaneswar| Lucknow|Patna|Bengaluru|Chennai|Vijayawada|Vizag |Tirupati | Kukatpally| Kolkata
:5: Pre GATE - 2017_EE

₹ 3240000 = ₹ 32.4 lakhs


Loss in the business = 10%
3240000
 Amount he received for making the flyover =  32400,000 = ₹ 3.24 crores
0.1

10. Study the following pie chart and table carefully to answer the following question that follow:
Percentage break up of employees working in various departments of an organisation and the ratio
of men to women in them.
Total number of employees = 1800
Percentage break up of employees:
Accounts
Dept.
Production
17% Dept.
28%
Marketing 18%
Dept
23% 14%

IT Dept HR Dept

Ratio of men to women


Department Men Women
Production 11 1
HR 1 3
IT 5 4
Marketing 7 5
Accounts 2 7
What is the number of men working in the marketing department?
(a) 132 (b) 174
(c) 126 (d) 189
10. Ans: (d)
18 7
Sol: Number of men working in the marketing department  1800    189
100 12

ACE Engineering Academy Hyderabad|Delhi|Bhopal|Pune|Bhubaneswar| Lucknow|Patna|Bengaluru|Chennai|Vijayawada|Vizag |Tirupati | Kukatpally| Kolkata


:6: Pre GATE - 2017_EE

Q.11 – Q.35 Carry one mark each.

11. The number of loops in the following signal flow graph is/are______

R(s) C(s)

Ans: 3 (Range 3 to 3)

12. Input voltage to the following single phase bridge rectifier is 220 V (RMS), 50 Hz. The load
resistance is 7 . The delay angle (in degree) required to deliver 3.61 kW is ____.

V0

220 V  R
50 Hz t
0    +  2

Ans:88 (Range 88.0 to 88.3)



1
 
Sol: V  2
or V 2
m sin 2 t dt

Vm2  1 

2     2 sin 2 

Vor2 V2  1 
P0   m      sin 2 
R 2R  2 

=
220 2       1 sin 2   3610
2

2  7  2 

1
      sin 2  1.64    88
2

ACE Engineering Academy Hyderabad|Delhi|Bhopal|Pune|Bhubaneswar| Lucknow|Patna|Bengaluru|Chennai|Vijayawada|Vizag |Tirupati | Kukatpally| Kolkata


:7: Pre GATE - 2017_EE

13. Lt
x 
x 2

 x  1  x  _________

Ans: 0.5

Sol: Lt x 2

 x  1  x  Lt  x  x  1  x 
2 x2  x 1  x 

 x  x  1  x 
x  x  2

1 0 1
 
1 0  0 1 2

14. The area frequency response characteristics of a 200MW, 50Hz power system is 105 MW/Hz and
regulation constant is 0.01Hz/MW. The load frequency constant of power system is
__________ MW/Hz
Ans: 5 (no range)
1
Sol: B  D 
R
1
105  D   D  5MW / Hz
0.01

15. The signal x(t) = cos(50t) + cos(80t) is sampled at 200Hz. The minimum number of samples
required to prevent leakage is _________

Ans: 40 (No Range)


1Ts 50 200 1 5
Sol:   
2 2 8 40
2 Ts 80 200 1 8
  
2 2 5 40
 The minimum number of samples required to prevent leakage is 40

ACE Engineering Academy Hyderabad|Delhi|Bhopal|Pune|Bhubaneswar| Lucknow|Patna|Bengaluru|Chennai|Vijayawada|Vizag |Tirupati | Kukatpally| Kolkata


:8: Pre GATE - 2017_EE

16. What is Vc1 0   =?


K
t=0
+ + + +
5V C1 = 10F C2 =20F 10V
– – – –

25 20
(a) V (b) V (c) 5V (d) None
3 3

Ans: (a)
5  10 10  20 5 20 25
Sol: Vc1 (0  )   =  
30 30 3 3 3

DISTRACTOR LOGIC:
10  20 20
Option: B Vc1 (0  )  
30 3
Option: C Vc1 (0  ) = Vc1 (0  ) =5 V
Option: D At t=0+ KVL is violated

17. A 3-, 460V, 1740 rpm, 60Hz, 4 pole WRIM has the following parameter per phase
R1 = 0.25 , R 12  0.2
X1  X12  0.5 , X m  30
The rotational losses are 1700 W with the rotor terminals short circuited. Find breakdown
frequency, when motor is operated from a 3-, 460V, 60Hz supply.

(A) 0.39 HZ (B) 11.5 HZ (C) 11.75 HZ (D) 26.44 HZ


Ans: (c)
0.2
Sol: STmax 
0.252  0.49  0.5
2

= 0.1958
f 2 T max  0.1958  60  11.752 Hz

ACE Engineering Academy Hyderabad|Delhi|Bhopal|Pune|Bhubaneswar| Lucknow|Patna|Bengaluru|Chennai|Vijayawada|Vizag |Tirupati | Kukatpally| Kolkata


:9: Pre GATE - 2017_EE

DISTRACTOR LOGIC
Option:A
0.2
Sol: STmax 
0.252  30  0.5
2

= 0.0065
f 2 T max  0.0065  60  0.39 Hz
Option B:
0.2
STmax 
0.25  0.49  0.5
2 2

= 0.1918
f 2 T max  0.1918  60  11.5 Hz
Option:D
0.45
STmax 
0.252  0.49  0.5
2

= 0.4407
f 2 T max  0 . 4407  60  26 . 44 Hz

18. An electrodynamometer wattmeter measures a power in single phase,50 Hz AC circuit. The load
voltage is 230V and load current is 10A at 0.5 lagging p.f. The wattmeter pressure coil resistance is
10k and inductance is 100mH, then % error will be
(A) zero (B) -1.68% (C) 0.54% (D) 0.156%
Ans: (C)
XP
Sol: tan  
RP ZP
XP
2  50  100  10 3 
 =0.00312
10  103 RP

 = 0.179
% error = + (tan tan)100
= 1.732 0.00312 100
= 0.54 %

ACE Engineering Academy Hyderabad|Delhi|Bhopal|Pune|Bhubaneswar| Lucknow|Patna|Bengaluru|Chennai|Vijayawada|Vizag |Tirupati | Kukatpally| Kolkata


: 10 : Pre GATE - 2017_EE

DISTRACTOR LOGIC
Option: (A) There is no effect of pressure coil resistance on wattemeter reading
Option: (B) if it is energey meter
sin(10  30)  0.5
% error =  100
0.5
= -1.68%
Option: (D) % error = + (cos Tan) 100
= 0.5  0.00312  100 = 0.156%

19. If A, B & C are n  n matrices and | A | = 2, | B | = 3 & | C | = 5 then the value of | A2 B C–1 | =?
6 12 18 24
(A) (B) (C) (D)
5 5 5 5
Ans: (B)
A A B 2  2  3 12
Sol: A 2 B C 1   
C 5 5

ACE Engineering Academy Hyderabad|Delhi|Bhopal|Pune|Bhubaneswar| Lucknow|Patna|Bengaluru|Chennai|Vijayawada|Vizag |Tirupati | Kukatpally| Kolkata


: 11 : Pre GATE - 2017_EE

20. In the amplifier circuit shown in figure, the transistor parameters with usual notations are
gm = 0.015S, rbe1  1K , rbb
1
 90 , C1be  20pF and C1bc  3pF . Neglecting the loading effect of
biasing resistors, R1 & R2, the mid-frequency voltage gain of the amplifier is
VCC

R2

C1

C2

INPUT R2
1 k OUTPUT

(A) – 13.76 (B) 0 (C) 0.936 (D) 1

Ans: (C)
Sol: VCC

R2

C1

C2

INPUT R2
1 k OUTPUT

Consider the parameters of BJT


rbe1  1K , rbb
1
 90 & gm = 0.015S,
Step(1) : h ie  rbe1  rbb
1
 1090
h fe
Consider, rbe1   h fe  rbe1  g m  1k  0.015 ℧ = 15
gm
Step(2): The given circuit is emitter follower (CC Amplifier)
The mid-frequency voltage gain in a CC amplifier,

AV 
1  h fe R E =
16 1K
=
16K
h ie  1  h fe R E 1.09K  16  1K 17.09K
AV = 0.936

ACE Engineering Academy Hyderabad|Delhi|Bhopal|Pune|Bhubaneswar| Lucknow|Patna|Bengaluru|Chennai|Vijayawada|Vizag |Tirupati | Kukatpally| Kolkata


: 12 : Pre GATE - 2017_EE

DISTRACTOR LOGIC
Option: A: If the given circuit is assumed as CE Amplifier, with a load resistance of RL=1k ; then,
Step(1) : h ie  rbe1  rbb
1
 1090
h fe
Consider, rbe1   h fe  rbe1  g m  1k  0.015 ℧ = 15
gm
h fe R L 15  1k
Step(2): AV= -   13.76
hie 1.09k
Option: B If the given circuit is assumed as CE Amplifier, with load resistance of Rc;
Step(1) : h ie  rbe1  rbb
1
 1090
h fe
Consider, rbe1   h fe  rbe1  g m  1k  0.015 ℧ = 15
gm
Step(2) : As Rc = 0 given in the circuit.
h fe R c 15  0
Av    0
hie 1.09k
Option : C: CC Amplifier; Av = 0.936
Option : D : The voltage gain in a cc amplifier ideally is ‘1’

21. The AC source in the circuit shown below has a voltage magnitude of 5 volts, which is divided
partly over the 1kΩ resistor and the remainder over the rest of the circuit. Compute the magnitude
of the voltage over the 1kΩ.

1k 2k

5 sin (t)
 4k 3k

1:3
(A) 0.1 V (B) 0.89 V (D) 1.08 V (D) 3.36 V

Ans: (D)
Sol: First, find the equivalent resistance looking into the primary (left) terminals of the transformer. The
resistance on the right side of the transformer is 5 kΩ since the 2 k and 3 k are in series. This is
2
1
then reflected to the left side by the transformer resistance equation: R eq    5k
3
ACE Engineering Academy Hyderabad|Delhi|Bhopal|Pune|Bhubaneswar| Lucknow|Patna|Bengaluru|Chennai|Vijayawada|Vizag |Tirupati | Kukatpally| Kolkata
: 13 : Pre GATE - 2017_EE

Req = 0.56 k Now we have the 4 kΩ and 0.56 kΩ resistors in parallel, which is 0.488 kΩ. The
0.488 kΩ is in series with the 1 kΩ , so we can use a voltage divider equation to find the voltage
 1k 
over the 1kΩ resistor: V   5V
 1k  0.488k 
V = 3.36 V
DISTRACTOR LOGIC
Option A: R eq  3  5k = 45 k
2

4 k series with 45 k and is equal to 49 k.


 1k 
V   5V = 0.1 V
 1k  49 k 
2
1
Option B: R eq    5k = 0.56 k
3
4 k in series with 0.56 k and is equal to 4.56 k.
 1k 
V   5V = 0.89 V
 1k   4 . 56 k  
Option C: R eq  3  5k = 45 k
2

4 k parallel with 45 k and is equal to 3.6 k.


 1k 
V   5V = 1.08 V
 1k  3.6 k 

22. A dual slope integrating type DVM is used to measure a voltage signal V(t) =
1
(100+100sin100t)V, in its (0-200)V range of operation and it has 3 - digit display. The
2
accuracy specification of this DVM is 0.5% of reading + 1 digit. What is the reading of this DVM
and percentage error in reading?
(A) 100 V, 0.6% (B) 100V, 1.5% (C) 70.7 V, 1.5% (D) 200V, 0.6%
Ans: (A)
Sol: DVM measures average value of input voltage signal.
 DVM reading = 100 V
1 digit = 1 count
= 1 step

ACE Engineering Academy Hyderabad|Delhi|Bhopal|Pune|Bhubaneswar| Lucknow|Patna|Bengaluru|Chennai|Vijayawada|Vizag |Tirupati | Kukatpally| Kolkata


: 14 : Pre GATE - 2017_EE

= r (resolution)
200
Resolution of DVM in 200V range is
2  10 3

= 0.1 V
0.5
Error =  100  0.1
100
= 0.6 V
0.6
% error in reading of 100V =  100%
100
= 0.6%

DISTRACTOR LOGIC
Option: (B) DVM measures average value of input voltage signal.
 DVM reading = 100 V
1 digit = 1 count
= 1% of error
0.5
%error =  100  1 = 1.5%
100
Option: (C) DVM measures RMS value of input voltage signal.
 DVM reading = 70.7 V
1 digit = 1 count
= 1% of error
0.5
%error =  100  1 = 1.5%
100
Option: (D) DVM reading =Vdc + Vm(ac)
=100+100
=200V
1 digit = 1 count = 1 step
= r (resolution)
Resolution of DVM in 200V range
200
=
2  103
= 0.1 V

ACE Engineering Academy Hyderabad|Delhi|Bhopal|Pune|Bhubaneswar| Lucknow|Patna|Bengaluru|Chennai|Vijayawada|Vizag |Tirupati | Kukatpally| Kolkata


: 15 : Pre GATE - 2017_EE

0.5
Error =  100  0.1
100
= 0.6 V
0.6
% error in reading of 100V =  100%
100
= 0.6%


23. The electric field intensity is given inside a sphere of radius Rbm as E  4R 2 r̂ (N/C). If the sphere
b
has permittivity ‘’, then the total electric displacement leaving the sphere R  m will be
2
(A) 4b4 Coulomb (B) b4 Coulomb
b 4
(C) b4 Coulomb (D) Coulomb
4
Ans: (B )

Sol: Given: E  4R 2 r̂ (N/C); R  b
 
D  E
= 4  R2 r̂ C/m2
From Gauss’s Law
 
 net  Q enc   D.dS
s

(or)
 net  D r  Area
= 4R2  4R2
= 16R4
b
The net electric flux leaving the sphere of radius R  is given by
2
4
b
 net  16 
2
 net = b4 C

ACE Engineering Academy Hyderabad|Delhi|Bhopal|Pune|Bhubaneswar| Lucknow|Patna|Bengaluru|Chennai|Vijayawada|Vizag |Tirupati | Kukatpally| Kolkata


: 16 : Pre GATE - 2017_EE

DISTRACTOR LOGIC
Option A: Simplification mistake
4
b
 net  16 
2
4
b
 16   4b 4
4
Which is wrong answer

Option C: If we take directly E , while applying
Gauss’s law, then net = 4R2  4R2
4
b
 16   b 4
2
Which is wrong answer
Option D: net = Dr  Area
2 b 4
If we take area = R then which results net  , which is wrong answer.
4
dy 4 dy3 dy 2 dy
24. The general solution of 4
 6 3
 12 2
8  0 is
dx dx dx dx
(A) y = C1 + (C2 + C3 x + C4 x2) e2x (B) y = (C1 + C2 x + C3 x2) e2x
(C) y = (C1 + C2 x + C3 x2 + C4 x3) e2x (D) y = C1 + C2 x + C3 x2 + C4 e2x
Ans: (A)
Sol: The given equation is (D4 – 6D3 +12D2 – 8D) y = 0
D(D3 – 6D2 + 12D – 8) y = 0
D(D – 2)3 = 0
 D = 0, 2, 2, 2
 The required solution is (A)

25. The Decimal equivalent of 2’s complement representation of 8 bit integer is most positive when
except _________
(A) MSB are zeros (B) LSB are zeros
(C) MSB are ones (D) LSB are ones

ACE Engineering Academy Hyderabad|Delhi|Bhopal|Pune|Bhubaneswar| Lucknow|Patna|Bengaluru|Chennai|Vijayawada|Vizag |Tirupati | Kukatpally| Kolkata


: 17 : Pre GATE - 2017_EE

Ans: (C)
Sol: If MSB of 2’s complement representation of integer is zero then it represents the positive number.
So option (C) is correct.
DISTRACTOR LOGIC
Option (A): If ‘Except’ is ignored in the given question, then option (A) is correct option.
Option (B): If LSB is considered as sign bit and ‘Except’ is ignored in the given question, then option
(B) is correct option.
Option (D): If LSB is considered as sign, then option (D) is correct option.

 e 1 / s  cos 2 t  ea / s 
26. L1  1 / 2   then L1  1 / 2  = ?
s  t s 
cos 2 at cos 2 t cos 2 at cos at
(A) (B) (C) (D)
at at t t
Ans: (C)
Sol: By using change of scale properly
  s 
L1 f    a F(at)
  a 
 
 1 
 e s / a   a cos 2 at
L1  1 

 s  2  at
  
 a  
 e  a / s  cos 2 at
 L1  1 / 2  
s  t

27. A 220V, 3- system supplies 363W to a wye connected balanced load at 0.8 pf lagging. The value
of phase current is
(A) 1.2–36.87 (B) 1.236.87 (C) 2–36.87 (D) 2+36.87

ACE Engineering Academy Hyderabad|Delhi|Bhopal|Pune|Bhubaneswar| Lucknow|Patna|Bengaluru|Chennai|Vijayawada|Vizag |Tirupati | Kukatpally| Kolkata


: 18 : Pre GATE - 2017_EE

Ans: (A)
Sol: ZL
220V
ZL
ZL

220
Phase voltage =
3
Total real power supplied by source
P = 3VphIphcos
P 363
I ph   = 1.2 A
3Vph cos   220 
3  0.8
 3
∵  = cos-1[0.8]=36.87

Lagging so, Iph = 1.2–36.87A


(option A is correct)
DISTRACTOR LOGIC
220
Option: B Phase voltage =
3
Total real power supplied by source
P = 3VphIphcos
P 363
I ph   = 1.2 A
3Vph cos   220 
3  0.8
 3
∵  = cos-1[0.8]=36.87

Lagging so, Iph = 1.236.87A


Option: C Phase voltage = 220 V
Total real power supplied by source
P = 3VphIphcos
P 363
I ph   2A
3Vph cos  220  0.8

∵  = cos-1[0.8]=36.87

Lagging so, Iph = 2-36.87A


ACE Engineering Academy Hyderabad|Delhi|Bhopal|Pune|Bhubaneswar| Lucknow|Patna|Bengaluru|Chennai|Vijayawada|Vizag |Tirupati | Kukatpally| Kolkata
: 19 : Pre GATE - 2017_EE

Option: D Phase voltage = 220 V


Total real power supplied by source
P = 3VphIphcos
P 363
I ph   2A
3Vph cos  220  0.8

∵  = cos-1[0.8]=36.87

Lagging so, Iph = 236.87A

28. A synchronous motor is operating on bus bar at no-load condition with normal excitation. If the
load on the motor is increased by keeping the field excitation same, for this
(A) the motor operates at lagging pf, absorbs reactive power
(B) the motor operates at leading pf, absorbs reactive power
(C) the motor operates at lagging pf, delivers reactive power
(D) the motor operates at leading pf, delivers reactive power
Ans: (A)
Sol: At normal excitation, no load condition  = 0, E = V
As load increases ‘’ increases then
Ecos < V  under excitation
 lagging pf, absorbs reactive power.

29. The controller shows in figure below is

R2
Vi(s) R1
1 
V0(s)
+

(A) proportional controller (B) PI controller


(C) PD controller (D) PID controller
Ans: (A)
V0 (s) R R
Sol: = TF = G(s) = (1) 2 ; 2 = constant
Vi (s) R1 R1
 It is a proportional controller

ACE Engineering Academy Hyderabad|Delhi|Bhopal|Pune|Bhubaneswar| Lucknow|Patna|Bengaluru|Chennai|Vijayawada|Vizag |Tirupati | Kukatpally| Kolkata


: 20 : Pre GATE - 2017_EE

30. For a lossless transmission line 50% series compensation and 50% shunt compensation is provided.
Then surge impedance loading (SIL) and maximum power transfer capacity (Pmax) will be
(A) SIL reduced by 1.732 times, Pmax increased by two times
(B) SIL remains same, Pmax increased by two times
(C) SIL increased by 1.732 times , Pmax remains same
(D) Both SIL and Pmax remain same

Ans: (B)
V2
Sol: SIL = ZCo is surge impedance with compensation
ZCo

1  K se
ZCo = ZC where ZC is surge impedance of uncompensated line Kse & Ksh are degree of
1  K sh
series and shunt compensations
From data Kse = 0.5, Ksh = 0.5
So, ZCo = ZC and no change in SIL
| VS || Vr | | Vs | Vr | |V |V |
Pmax = = = s r
| B| Zc sin   Z c 
| Vs | Vr
=
X line
With 50% series compensation,
| Vs | Vr |
Pmax new =
X line  0.5X line
| Vs | Vr |
= 2.
X line
Pmax increased by 2 times
DISTRACTOR LOGIC
V2
Option A: SIL  , ZC0 ; is surge impedance with compensation
Z C0

1  K se
Z C 0  Zc ; where Zc is surge impedance, and series and shunt compensations from data
1  K sh
Kse = 0.5; Ksh = 0.5

ACE Engineering Academy Hyderabad|Delhi|Bhopal|Pune|Bhubaneswar| Lucknow|Patna|Bengaluru|Chennai|Vijayawada|Vizag |Tirupati | Kukatpally| Kolkata


: 21 : Pre GATE - 2017_EE

1  0 .5
Z C 0  Zc
1  0 .5
0 .5
 ZC
1 .5
1
=  ZC
3
Reduced by 1.732 times
| Vs || Vr | | Vs | Vr | | Vs | Vr | | VS | Vr |
Pmax    
| B| ZC sin   Z C  X line
With 50% series compensation, Pmax,new
| VS || Vr |
=
X line  0.5X line
| VS || Vr |
=2
X line
Pmax increased by 2 times
V2
Option C: SIL  , ZC0 is surge impedance with compensations
Z C0

1  K sh
Z C 0  Zc ; Where ZC is surge impedance, and series and shunt compensation
1  K se
from data Kse = 0.5; Ksh = 0.5
1  0 .5
Z C 0  ZC
1  0 .5
 ZC 3
Increased by 1.732 times
| Vs || Vr | | VS || Vr |
Pmax  
| B| ZC sin  
| VS || Vr |

Z C 
| Vs || Vr

X line  K se X line  K sh X line
| VS || Vr |

X line  0.5X line  0.5X line
ACE Engineering Academy Hyderabad|Delhi|Bhopal|Pune|Bhubaneswar| Lucknow|Patna|Bengaluru|Chennai|Vijayawada|Vizag |Tirupati | Kukatpally| Kolkata
: 22 : Pre GATE - 2017_EE

| Vs || Vr |

X line
Pmax remains same
V2 1  K se
Option D: SIL  , ZC0 is surge impedance with compensation Z c0  ZC ; where ZC is surge
Z C0 1  K sh
impedance of uncompensated line, and Kse & Ksh are degree of series and shunt
compensation from data Kse = 0.5 ; Ksh = 0.5
1  0 .5 Z C
ZC
 1
1  0 .5
So, ZC0  ZC no change in SIL
| Vs || Vr | | VS || Vr |
Pmax  
| B| ZC sin  
| VS || Vr |

Z C 
| Vs || Vr

X line  K se X line  K sh X line
| VS || Vr |

X line  0.5X line  0.5X line
| Vs || Vr |

X line
Pmax remains same

31. In a uniform region of free space, if the electric field intensity is given by E  x̂ 4x  ŷ3y Volt/m,
then the potential at X(0, 2, 2)m with respect to Y(2, 2, 2)m is
(A) 8V (B) 8V (C) 16 V (D) 16 V
Ans: (A )
Sol: Potential at X w.r.t Y is given by
 
X ( 0, 2, 2 )

VXY    .d 
E
Y ( 2, 2, 2 )

0 2

    4xdx   3ydy
2 2 

ACE Engineering Academy Hyderabad|Delhi|Bhopal|Pune|Bhubaneswar| Lucknow|Patna|Bengaluru|Chennai|Vijayawada|Vizag |Tirupati | Kukatpally| Kolkata


: 23 : Pre GATE - 2017_EE

2 0
x
 VXY = – 4  8V
2 2

DISTRACTOR LOGIC
  Y
Option B: If we take Vxy = –  E. d 
X

Then VXY = – 8 V which is wrong answer .


2 2

Option C: VXY     4 xdx   3ydy
0 2 
2
 4 x 2 = – 16V, which is incorrect answer .
0

0 2

Option D: VXY     4 xdx   3ydy
2 2 
0
 4 x 2 = 16V, it is a simplification mistake in integration .
2

Z Z12 
32. A two port network having Z-parameters as Z   11 is terminated to a load impedance ZL
 Z 21 Z 22 
as shown. The input impedance Zin of two port network is given by

Zin Two-port
ZL
N/W

(A) Z11  Z12 Z 21 (B) Z11  Z12 Z 21  Z L  (C) Z11 + ZL (D) Z22 +ZL
Z 22  Z L Z 22

Ans: (A)
V1
Sol: Input impedance, Zin 
I1 I1 I2
+ +
Z –parameter equations V1 V2 ZL
– –
V1 = Z11I1 +Z12I2
V2 = Z21I1 +Z22I2
But, V2 = –I2ZL
So,

ACE Engineering Academy Hyderabad|Delhi|Bhopal|Pune|Bhubaneswar| Lucknow|Patna|Bengaluru|Chennai|Vijayawada|Vizag |Tirupati | Kukatpally| Kolkata


: 24 : Pre GATE - 2017_EE

–I2ZL = Z21I1 +Z22I2


–I2(ZL+Z22) = Z21I1
Z 21
I2   .I1
Z L  Z 22
Z 21Z12
So, V1  Z11I1  .I1
Z L  Z 22

V1  Z .Z 
So, Zin    Z11  21 12 
I1  Z L  Z 22 
DISTRACTOR LOGIC
V1
Option: B Input impedance, Zin  I1
I1 I2
+ +
Z –parameter equations V1 V2 ZL
– –
V1 = Z11I1 +Z12I2
V2 = Z21I1 +Z22I2
But, V2 = –I1ZL
So,
–I1ZL = Z21I1 +Z22I2
I1(Z21+ZL) = Z22I2
Z21  ZL
I2  .I1
Z22
Z12  ( Z 21  Z L )
So, V1  Z11I1  .I1
Z 22

V1  Z  ( Z12  Z L ) 
So, Zin    Z11  21 
I1  Z 22 
Option: C If two networks are in cascade than
Z eq  Z A   Z B 
V1 A B
Z Z12   Z L Z L   Z11  Z L Z12  Z L  V2
  11  
 Z 21 Z 22   Z L Z L   Z 21  Z L Z 22  Z L 

The input impedance is given by


V1
 Z11  Z L
I1

ACE Engineering Academy Hyderabad|Delhi|Bhopal|Pune|Bhubaneswar| Lucknow|Patna|Bengaluru|Chennai|Vijayawada|Vizag |Tirupati | Kukatpally| Kolkata


: 25 : Pre GATE - 2017_EE

Option: D If two networks are in cascade than


Z eq  Z A   Z B 
Z Z12   Z L Z L   Z11  Z L Z12  Z L  V1 A B V2
  11  
 Z 21 Z 22   Z L Z L   Z 21  Z L Z 22  Z L 

The input impedance of output port is given by


V2
 Z 22  Z L
I2

33. A 4 pole series motor has 944 wave connected armature conductors. At a certain load the flux/pole
is 34.6 mWb and the total mechanical torque developed is 209 N-m. With an applied voltage of
500V, the speed of the motor will be (The total motor resistance is 1.)
(A) 422 rpm (B) 440 rpm (C) 477 rpm (D) 496 rpm

Ans: (B)
1 ZIa P
Sol: T  
2 A
1 34.6  103  944  Ia  4
 209  
2 2
Ia = 20.10 A
Eb = V –IaRa = 500 – 20.10 1=479.89 A
ZNP
Eb   479.89
60A
34.6  103  944  N  4

60  2
 N  440.77 rpm

DISTRACTOR LOGIC
Distractor logic:
1 ZIa P
Option: A T  
2 A
1 34.6  103  944  Ia  4
 209  
2 4
Ia = 40.20 A

ACE Engineering Academy Hyderabad|Delhi|Bhopal|Pune|Bhubaneswar| Lucknow|Patna|Bengaluru|Chennai|Vijayawada|Vizag |Tirupati | Kukatpally| Kolkata


: 26 : Pre GATE - 2017_EE

Eb = V + IaRa = 500 – 40.20 1= 459.8 A


ZNP
Eb   459.8
60A
34.6  103  944  N  4
459.8 
60  2
 N  422.32 rpm

1 ZIa P
Option: C T  
2 A
1 34.6  103  944  Ia  4
 209  
2 2
Ia = 20.10 A
Eb = V + IaRa = 500 + 20.10 1= 520.1 A
ZNP
Eb   520
60A
34.6  103  944  N  4
520 
60  2
 N  477.61 rpm

1 ZIa P
Option: D T  
2 A
1 34.6  103  944  Ia  4
 209  
2 4
Ia = 40.20 A
Eb = V + IaRa = 500 + 40.20 1= 540.2 A
ZNP
Eb   540.2
60A
34.6  103  944  N  4
540.2 
60  2
 N  496.16 rpm

ACE Engineering Academy Hyderabad|Delhi|Bhopal|Pune|Bhubaneswar| Lucknow|Patna|Bengaluru|Chennai|Vijayawada|Vizag |Tirupati | Kukatpally| Kolkata


: 27 : Pre GATE - 2017_EE

34. The below program is executed by 8085 microprocessor


XRA A
MOV L, A
MOV H, L
DCX H
DAD H
MOV M, A
INR M
MOV A, M
After execution, what will be the contents of PSW
(A) FFFEH (B) 0101 H (C) 01FFH (D) FE01H
Ans: (B)
Sol: (A) = 00H [(A)  (A) = 00H]
 cy =0, P = 1, AC = 0, Z = 1, S = 0
(L)  (A) = 00H
(H)  (L) = 00H
DCX H  0000H – 1 = FFFFH
FFFF = 1111 1111 1111 1111
FFFF = 1111 1111 1111 1111
1111 1111 1111 1110  FFFEH
(HL) = FFFEH with cy = 1
Other flags are remains same.
(FFFEH)  (A) = 00H
00H+1 = 01H
= (FFFEH)
Cy flag not affected for INR  cy remains 1
Remaining 4 flags are affected based on result
01H = 0000 0001  P = 0, AC = 0 , Z = 0, S = 0
(A)  (FFFEH) = 01H
(A) = 01H , (flag reg) = 00  0  0  1
= 0000 0001
= 01H
(PSW) = 0101H
ACE Engineering Academy Hyderabad|Delhi|Bhopal|Pune|Bhubaneswar| Lucknow|Patna|Bengaluru|Chennai|Vijayawada|Vizag |Tirupati | Kukatpally| Kolkata
: 28 : Pre GATE - 2017_EE

35. The output Z is

A
B

Z
A
B

(A) A  B (B) A  B (C) 1 (D) 0


Ans: (D)
Sol:

A
B
X
Z
Y
A
B

X=AB
Y=AB
Z = A B A  B
=AAB B
=00
Z=0

ACE Engineering Academy Hyderabad|Delhi|Bhopal|Pune|Bhubaneswar| Lucknow|Patna|Bengaluru|Chennai|Vijayawada|Vizag |Tirupati | Kukatpally| Kolkata


: 29 : Pre GATE - 2017_EE

DISTRACTOR LOGIC
Option (C): If A is takn as A
X=AB
Y=AB
Z = A B  A  B
=AABB
=01
Z=1
Option (A): If XOR gate is taken OR-gate
Z = [A  B] + [A  B]
=AB
Optio (B): If X = A  B
Y=AB
And output gate considers as OR gate then
Z= A B  A  B
=AB

ACE Engineering Academy Hyderabad|Delhi|Bhopal|Pune|Bhubaneswar| Lucknow|Patna|Bengaluru|Chennai|Vijayawada|Vizag |Tirupati | Kukatpally| Kolkata


: 30 : Pre GATE - 2017_EE

Q.36 – Q.65 carry two marks each

36. Consider the system shown in figure below

1
+
 s(s  1)

The approximate 2% settling time of the response to a unit step input is __________ seconds.
Ans: 8 (Range 7.9 to 8.3)
1
Sol: CE = 1 + =0
s(s  1)
s2 + s + 1 = 0
n = 1 and 2n = 1
n = 0.5
4
ts = = 8 sec for 2% criterion
n

37. Input voltage to the following DC-DC converter is 20 V and output voltage is 50 V. Duty cycle
ratio is 0.5. RMS value of diode current (in ampere) is ____.

20 H
+

Vdc + C 50  V0

ACE Engineering Academy Hyderabad|Delhi|Bhopal|Pune|Bhubaneswar| Lucknow|Patna|Bengaluru|Chennai|Vijayawada|Vizag |Tirupati | Kukatpally| Kolkata


: 31 : Pre GATE - 2017_EE

Ans: 2 (Range 2 to 2)
Sol: iL
6A
 30
20
L
L
t
DT T T

6A

( – D)T t

Vdc 20
In continuous conduction mode, V0   = 40V
1  D 1  0.5
But given V0 > 40 V, so it is discontinuous mode of operation.
Power balance equation P0 = Pin
 50 
 Vdc .I s  50   
 50 
50
Is   2.5 A
20
V0  50
   2.5
Vdc   D 20
 = 2.5   – (2.5 0.5)
 = 0.833
1
 I L Max  T
IL  2 =2.5
T
IL Max = 6 A
1
 62  5 1  T  2
ID,rms =       
 3 6 2 T 
1
 62 2  2
  
 3 6
ID,rms =2 A

ACE Engineering Academy Hyderabad|Delhi|Bhopal|Pune|Bhubaneswar| Lucknow|Patna|Bengaluru|Chennai|Vijayawada|Vizag |Tirupati | Kukatpally| Kolkata


: 32 : Pre GATE - 2017_EE

38. Region 1, where r1 = 4, is the side of the plane y + z =1 containing the origin (shown in figure). In

region 2,r2 = 6. If the magnetic flux density in region 1 is B1  2â x  â y (Tesla), then the
magnitude of magnetic flux density (in T) in region 2 is ______.

z
â n
(2)
r2=6
o y
(1)
r1=4

Ans: 3.2 (Range: 3 to 3.5)


Sol: The unit vector normal to the plane y + z = 1 is given by
â y  â z
â n 
2
  â  â z 
B n1  B1.â n = (2â x  â y ). y 
 2 
1
Bn1=
2
 1  â y  â z 
B n1  B n 1 â n   
2  2 

B n1  0.5â y  0.5â z

B n 2  B̂ n 1  0.5â y  0.5â z
  
B t1  B1  B n 1

 2â x  â y   0.5â y  0.5â z 



B t1  2â x  0.5â y  0.5â z
  
B t1 B t 2

3

 B t 2   2 B t1    2â x  0.5â y  0.5â z 
1 2  1  2

B t 2  3â x  0.75â y  0.75â z
 
  
B 2  3â x  1.25â y  0.25â z  B 2  B t 2  B n 2 

 B2  32  1.252   0.252  3.259 Tesla
ACE Engineering Academy Hyderabad|Delhi|Bhopal|Pune|Bhubaneswar| Lucknow|Patna|Bengaluru|Chennai|Vijayawada|Vizag |Tirupati | Kukatpally| Kolkata
: 33 : Pre GATE - 2017_EE

39. The maximum power transferred to R is _____(in watts).


10

20V R 10

Ans: 5
Sol: Rth = 10||10=5
Vth = 10V
Vs2 10 2 100
Pmax     5W
4.R L 4  5 20

40. The three-winding ideal transformer in the following figure has N1  N 2  2 N3 and identical load
resistors (R ) connected across coils 2 and 3. The input impedance Z1 as indicated in the figure is
________R .
R

I2 +V2 –

I1
+ N2 –
V1  N1 N3 V3 R
– +

I3
Z1

Ans: 0.8 (Range 0.8 to 0.8)


Sol: MMF balance requires that
N1 I1  N 2 I 2  N 3 I 3
1
N1I1  N1I 2  N1I3
2
Or
1
I1  I 2  I 3 …………. (1)
2
Since the value of flux through all three coils is identical, V1  V2  2V3 .

ACE Engineering Academy Hyderabad|Delhi|Bhopal|Pune|Bhubaneswar| Lucknow|Patna|Bengaluru|Chennai|Vijayawada|Vizag |Tirupati | Kukatpally| Kolkata


: 34 : Pre GATE - 2017_EE

V2 V1
By Ohm's law, I 2   ……….. (2)
R R
V3 V1
I3   …………… (3)
R 2R
Use (2) and (3) in (1) to find
V1 V1 5V1
I1   
R 4R 4R
Hence,
V1 4
Z1   R
I1 5

41. The current I (in Amp) is _____.

5A
+
10V N 5V

2 N 4A

Ans: 1
V21 I12
Sol: 
V1 I1

5 I12

10 4 I
2A 2 2
20
I12   2A
10
10
R th   2
5
I = 1Amp

ACE Engineering Academy Hyderabad|Delhi|Bhopal|Pune|Bhubaneswar| Lucknow|Patna|Bengaluru|Chennai|Vijayawada|Vizag |Tirupati | Kukatpally| Kolkata


: 35 : Pre GATE - 2017_EE

42. Synchronous generator is connected to an infinite bus by a transformer and lossless network.
Infinite bus voltage is 1.0 pu and a voltage of generator is 1.2 pu. The transfer admittance is 2.0 pu.
The power transfer by the generator is 1.0 pu. A 3-phase fault is taking place in the lossless
network so that the power transfer is zero. The fault is cleared by circuit breaker and the original
network is restored. The critical clearing angle made by the rotor at the time fault gets cleared by
circuit breaker is _____ (in degrees)

Ans: 87.6 (Range: 86 to 88)


Sol: Ps = Pe = 1.0
EV
PM1   (EV) y eq  2.0 (1.2  1.0)  2.4
X1eq

 P 
 0  sin 1  S 
 PM 
 1 
 1.0 
 sin 1  ele. deg ree = 24.62
 2.4 

 0 ( rad )   0  =0.429
180
Fault: Pe2  0, Pm2  0

 P 
 m  180  sin 1  S 
 Pm 
 3 
Pm3  Pm1

 1.0 
 m  180  sin 1   = 180 – 24.62= 155.38
 2.4 

 m ( rad )   m  = 2.711
180
 PS  m   e   Pm3 cos  m 
 c  cos 1  
 Pm3 
1.0 m   o   2.4 cos  m 
 C  cos 1   ele. deg ree
 2 .4 

1.02.711  0.429  2.4 cos155.38 


 c  cos 1   = 87.6
 2.4

ACE Engineering Academy Hyderabad|Delhi|Bhopal|Pune|Bhubaneswar| Lucknow|Patna|Bengaluru|Chennai|Vijayawada|Vizag |Tirupati | Kukatpally| Kolkata


: 36 : Pre GATE - 2017_EE

43. A 3-phase synchronous motor has a reactance of 0.8 pu with negligible resistance. When connected
to busbar at rated voltage and the excitation adjusted to an emf of 1.2 pu, the machine draws rated
input kVA. The mechanical power developed by the motor is ______ pu.

Ans: 0.992 (Range: 0.8 to 1)


Sol: Xs = 0.8 pu, E = 1.2 pu, V = 1.0 pu, Ia = 1pu
I a z s  E 2  V 2  2VE cos 

 1 0.8 = 1.2 2  12  2  1  1.2 cos 


  = 41.4
EV 1.2  1
P sin   sin 41.4 o
Xs 0.8
= 0.992 pu

44. A 250 V shunt motor has an armature resistance of 0.5  and field resistance of 250. When
driving a load at 600 rpm, the torque of which is constant, the armature takes 20 A current. If it is
desired to raise the speed from 600 to 800 rpm, then the resistance to be inserted in the shunt field
circuit is ________.

Ans: 88.0 to 88.5


Sol: Torque constant T2 = T1
2 I a 2  1I a 1
 Ish 2 Ia 2  Ish1 Ia 1

V 250
Ish 1    1A
R sh 250
Ish 2 Ia 2  1  20
20
Ia 2 
Ish 2
E b1  V  Ia 1 R a  250  20  0.5  240 A
20 10
E b 2  V  I a 2 R a  250   0.5 = 250 
Ish 2 Ish 2

ACE Engineering Academy Hyderabad|Delhi|Bhopal|Pune|Bhubaneswar| Lucknow|Patna|Bengaluru|Chennai|Vijayawada|Vizag |Tirupati | Kukatpally| Kolkata


: 37 : Pre GATE - 2017_EE

10
250 
Eb2 Ish 2  N 2 Ish 2 Ish 2  800
  
E b1 Ish 1  N1 240 1  600
10
= 250   320 Ish 2
Ish 2

 32 Ish2 2  25 Ish 2  1  0

25  252  4  32  1
I sh 2 =
2  32
I sh 2 = 0.739 A or (0.0422 A is too low)
V 250
Ish 2  0.739A  
R sh 2 R sh 2
250
R sh 2   338.29 
0.739
Resistance to be added
= 338.29–250 = 88.29 

45. In a power system network Thevenin’s impedance and voltage with respect to a bus are given as
0.9 pu and j0.2 pu respectively. The load kept at that bus consumes constant complex power of 1 +
j1 pu. A shunt capacitor bank is connected at that bus to increase voltage magnitude to 1 pu. The
power factor of load capacitor combination is ____________ (lag)
Ans: 0.894 (Range: 0.85 to 0.92)
Sol: From given data Vth = 0.9 pu; Zth = j0.2 pu. To increase voltage magnitude to 1 pu, a shunt
capacitor bank was connected.
Vc = 1pu
Zth
ZC
VC= Vth +
ZC  Zth +
Vth VC ZC
ZC
1 = 0.9 –
j0.2  ZC –

So, ZC = j2 pu

Load at that bus is 1 + j1 pu

ACE Engineering Academy Hyderabad|Delhi|Bhopal|Pune|Bhubaneswar| Lucknow|Patna|Bengaluru|Chennai|Vijayawada|Vizag |Tirupati | Kukatpally| Kolkata


: 38 : Pre GATE - 2017_EE

| V |2 1 PS +jQs
From this, Qsh cap =  = 0.5 pu
Xc 2
So, PS = Pload = 1 pu
QS = Qload  Qsh cap = 0.5 pu PLoad = 1pu
Qshcap
Overall power factor QLoad = 1pu
Zc = – j2pu
Ps
coss=
Ps2  Qs2
1
= lag
1  (0.5) 2
= 0.894 lag

46. A three phase voltage source inverter (VSI) as shown in figure is feeding a star connected inductive
load of (0 + j20) /ph. If it is fed from a 650 V battery and operates with 180 conduction mode
with fundamental frequency of output as 50 Hz, the peak value of per phase load current in ampere
is ____________.

A
+
0
j20
650V A
DC B
C 0
j20
j20
0
B
- C

Ans: 22.69 (Range 22.6


V
to 22.8)
AN

Sol:
2Vdc/3

0 Vdc/3
t

T/6 T/6 T/6 T/6 T/6 T/6 T/6


T
D
iAN
B
E
0
A C F t

ACE Engineering Academy Hyderabad|Delhi|Bhopal|Pune|Bhubaneswar| Lucknow|Patna|Bengaluru|Chennai|Vijayawada|Vizag |Tirupati | Kukatpally| Kolkata


: 39 : Pre GATE - 2017_EE

2Vdc BC 2Vdc T V
ABC,   BC    dc
3L AC 3L 12 18Lf
Vdc DE V T V
Is BDE,   DE  dc   dc
3L BE 3L 6 18Lf
Vdc V
i 0 ( peak )  BC  DE   dc
18Lf 18Lf
Vdc
=
9Lf
But 2 Lf = 20
 20 
so Lf   
 2 
650  2
i0, peak =  22.69 A
9  20

47. The output voltage V0 of op-amp circuit shown in figure assuming op-amp & diodes as ideal, is
________________ V.
5k

5v 5k 200 k
– D1 D2 200 k
A1 –
+ A2
+ V0
2.5 k -10V 400k
80 k

Ans: V0 = 10 (Range: 10 to 10)


Sol: Step (1): For the given input of 5v, D1 is off & D2 is ON
5k
V01   5v  5v
5k
Step (2): KCL at the inverting input, V2 of op-amp2(A2)
5K
200K
5K D1
– 200K
5V A1 -
+ D2 V01 V2 A2 V0
Op-amp 2 (A2) +
2.5K
V01 10v V -10V
  0 0 400K 80K
200k 400k 200k

ACE Engineering Academy Hyderabad|Delhi|Bhopal|Pune|Bhubaneswar| Lucknow|Patna|Bengaluru|Chennai|Vijayawada|Vizag |Tirupati | Kukatpally| Kolkata


: 40 : Pre GATE - 2017_EE

 5V 10V 
V0 = 200k   =5V+5V
 200k 400k 
V0 =10 V

4 2

  x 
 y 2 dxdy becomes
2
48. By changing the order of interaction
1 y
2 x2 4 x2

  x  y dydx   x  y 2 dydx
2 2 2
(A) (B)
1 1 1 1
2 1 4 1

  x  y 2 dydx    x 
 y 2 dydx
2 2
(C) (D)
1 x2 1 x2

Ans: (A)
Sol: In the given double integral the limits of x : y  2 and y : 1 → 4
 The required area is shaded below
x y
y=4

y=1

x=2

4 2 2 x2
  x 
 y dxdy becomes   x 
 y 2 dydx
2 2 2

1 y 1 1

ACE Engineering Academy Hyderabad|Delhi|Bhopal|Pune|Bhubaneswar| Lucknow|Patna|Bengaluru|Chennai|Vijayawada|Vizag |Tirupati | Kukatpally| Kolkata


: 41 : Pre GATE - 2017_EE

49. In the circuit shown in figure , a silicon transistor with VBE = 0.7V,  = 100 is used. The transistor
is biased at _____ ?
10V

5 k IC

1 k

–10V
Fig.

(A) 9.2 mA (B) 2.1 mA (C) 1.86 mA (D) 0

Ans: (B)
10V
Sol: Step (1): KVL for BE loop of BJT
IC
0 – 0.7V –IE 1K+10V = 0 …….. (1) 5 k

9.3V
 IE   9.3mA …….. (2) [i.e JE is FB]
1K
+
  
 I C   I E  9.2mA ….. (3) VBE –

1  
1 k
Step (2): KVL for C-loop
10V – IC 5K –VC = 0 …… (4) –10V

VC = 10V – 9.2mf5K = – 36V ……. (5)


 VCB  VC  VB  36V  0  36V …… (6)

NOTE: ∵VCB is –Ve, collector junctions is F.B


10V
 BJT is operated in saturation region
5 k IC
Step (3): ∵ BJT is in saturation, VCEsat  0.2V
VC = – 0.5V
 VC  VCEsat  VE  0.5V ……. (1) VCE= 0.2V

KVL for collector –loop:


VE = – 0.7V
10V  (0.5V)
IC   2.1mA …. (2) 1 k
5K
–10V

ACE Engineering Academy Hyderabad|Delhi|Bhopal|Pune|Bhubaneswar| Lucknow|Patna|Bengaluru|Chennai|Vijayawada|Vizag |Tirupati | Kukatpally| Kolkata


: 42 : Pre GATE - 2017_EE

DISTRACTOR LOGIC
Option: A: If the device(BJT) is in forward active region, KVL for BE loop of BJT
0 – 0.7V – IE 1K+10V = 0
9.3V
IE   9.3mA
1K
   100
 I c   I E   9.3mA
1   101
Ic = 9.2mA
Option: B: Ic =2.1mA …… (1) (∵ Device is actually biased in saturation region)

Option:C: If the device is in inverse (or reverse) active region, + 10V


(i.e) E-B junction is R.B & C.B junction is F.B Ic
5 k
Assuming VCB = 0.7V  Vc  0.7V …… (1)
+ Vc = 0.7V
VCB
[∵VCB =VC–VB = VC] –
KVL for collector loop of BJT
10V –Ic 5K – 0.7V = 0 …… (1)
1 k
9.3V
Ic   1.86mA …… (2)
5K –10V
Option: D If the device is in cutoff region
IB = 0  IC = 0 …… (1)

50. The incidence matrix of a graph is given below


 1 0 1 1  1 0 
A   1  1 0 0 0  1
 0 1  1  1 1 0 
The number of possible Trees are
(a) 3 (b) 7 (c) 11 (d) 14
Ans: (b)
Sol: Given is reduced incidence matrix, so, Number of trees = det|[Ar][Ar]T|
 1 1 0 
 0 1 1 
 1 0 1 1  1 0   
   1 0  1
A r A r T   1  1 0 0 0  1  
1 0  1
 0 1  1  1 1 0  
 1 0 1 
 
 0  1 0 

ACE Engineering Academy Hyderabad|Delhi|Bhopal|Pune|Bhubaneswar| Lucknow|Patna|Bengaluru|Chennai|Vijayawada|Vizag |Tirupati | Kukatpally| Kolkata


: 43 : Pre GATE - 2017_EE

 4  1  3
A r A r     1 3  1
T

 3  1 4 
det A r A r   412  1  1 4  3  31  9
T

= 44–7 – 30 = 7 trees only

ACE Engineering Academy Hyderabad|Delhi|Bhopal|Pune|Bhubaneswar| Lucknow|Patna|Bengaluru|Chennai|Vijayawada|Vizag |Tirupati | Kukatpally| Kolkata


: 44 : Pre GATE - 2017_EE

51. Input voltage to the following Buck-Boost converter is 20 V. Assume that the capacitor is large to
treat output voltage is constant of 60 V. The switch is operating at 50 kHz with a duty ratio of 0.6.
Average value of inductor current is _____.

Vdc + IL L R V0

20 H
+

(A) 12 A (B) 7.2 A (C) 4.8 A (D) insufficient data

Ans: (C)
Sol:

Vdc + IL L R V0

20 H
+

 D  0.6
In CCM Buck Boost, V0  Vdc    20   30 V
1  D  0.4
But given V0 > 30 V, so it is discontinuous mode of operation.

iL

IL (max)
 V
0
V dc
L
L
t
DT T T

From iL waveform
20 60
DT  T  DT 
L L
4
 4 DT  3T     D  0.8
3
Vdc 20
 I L max   DT   0.6  20  12 A
L 20

ACE Engineering Academy Hyderabad|Delhi|Bhopal|Pune|Bhubaneswar| Lucknow|Patna|Bengaluru|Chennai|Vijayawada|Vizag |Tirupati | Kukatpally| Kolkata


: 45 : Pre GATE - 2017_EE

1
 12  0.8  20 
IL  2  4 .8 A
20 
( avg )

DISTRACTOR LOGIC
 D  0.6
Opton (A) : V0  Vdc    20   30 V
1  D  0.4
20 30
DT   T  DT 
L L
 =1 so it is continuous conduction mode
iL

IL (max)
 V
0
V dc
L
L

DT T t

Vdc
i L peak   DT
L
20
=  0.6  20
20
=12 A
 iL (avg) = 12 A
 D  0.6
Option B: V0  Vdc    20   30 V
1  D  0.4

iL

IL (max)
 V
0
V dc
L
L
t
DT T T

From wave form


20 60
DT  T  DT 
L L
4
 4 DT  3T     D  0.8
3

ACE Engineering Academy Hyderabad|Delhi|Bhopal|Pune|Bhubaneswar| Lucknow|Patna|Bengaluru|Chennai|Vijayawada|Vizag |Tirupati | Kukatpally| Kolkata


: 46 : Pre GATE - 2017_EE

Vo 30
 I L max   DT   0.6  20  18 A
L 20 
1
 18  0.8  20 
IL  2  7 .2 A
20 
( avg )

Option D: To check whether it is continuous or discontinuous condition, R value is required. But it is


not given in the problem, so it is insufficient data.
d2y  3
52. The solution of  y which passes through the origin and  n 2,  is ______
dx 2  4

(A) y 
ex
2
 e x (B)
8

3 x
e  e x  1
(C) y   e x  e  x 
2
(D)
ex
2
 e x

Ans: (C)
Sol: The given equation is (D2 – 1) y = 0
i.e., D =  1 are the roots of A. E
 y = (C1ex + C2e–x)
If it passes through the origin i.e. x = 0, y = 0
then C1 + C2 = 0 ………… (1)
 3
Similarly if passes through  n 2, 
 4
3
then   2C1  0.5C2  ………. (2)
4
By solving (1) & (2) for C1 ; C2
1 1
We get C1  & C2 
2 2
1 e x  e  x 
y is the required solution.
2

ACE Engineering Academy Hyderabad|Delhi|Bhopal|Pune|Bhubaneswar| Lucknow|Patna|Bengaluru|Chennai|Vijayawada|Vizag |Tirupati | Kukatpally| Kolkata


: 47 : Pre GATE - 2017_EE

53. The four arms of a wheatstone bridge are as follows: Arm AB =100 , BC = 10, CD = 4 and
DA = 50.The galvanometer has a resistance of 20 and is connected across BD. A source of 10V
DC is connected across AC, then the current through the galvanometer is
(A) zero (B)13.3mA (C)5.18mA (D)39mA

Ans: (C)
Sol: Vth = VB – VD B

 10 4  10
 10   10   100 Thevenin’s eq ckt
 10  100 4  50  G
Rth
A R C 
= 0.9091 – 0.7407 20 Ig

= 0.17V Vth + Rg= 20 G


50 4 –
100  10 50  4
R th   D
100  10 50  4
= 9.09091+3.7037 10V

= 12.7946
Vth 0.17
Ig    5.18mA
R th  20 12.7946  20

DISTRACTOR LOGIC
Option: (A) if bridge is balanced, ig = 0
Option: (B) if galvanometer resistance is neglected
Rth=12.79 

0.17V ig

0.17
ig  = 0.01329 A = 13.3 mA
12.179
Option: (D) if AB & AD branches are interchanged
10  10 10  4
Vth  
10  50 4  100
= 1.28 Volts
10  50 4  100
R th  
10  50 104

ACE Engineering Academy Hyderabad|Delhi|Bhopal|Pune|Bhubaneswar| Lucknow|Patna|Bengaluru|Chennai|Vijayawada|Vizag |Tirupati | Kukatpally| Kolkata


: 48 : Pre GATE - 2017_EE

= 12.18 
1.28
ig 
20  12.18
= 0.03977 A
= 39 mA

10
54. The Loop transfer function of the unity feedback system is then gain margin of the system
(s  2)
is
(A) 0 (B) 0.2 (C) 5 (D) 

Ans: (D)
Sol: The Nyquist plot (from  = 0 to  = ) is shown in figure below

Img

5 Real
= =0

Nyquist plot does not intersection the negative real axis


 Gain margin is infinite

DISTRACTOR LOGIC:
1
Option A: Point of intersection is ‘’, then GM = =0

 10  
Option B:   = tan1 2 = 180 |p  pc = 0
 j  2 
10 10
= =5
jpc  2 2
1
GM = = 0.2
5
10
Option C: 5
jpc  2
pc

 GM = 5
ACE Engineering Academy Hyderabad|Delhi|Bhopal|Pune|Bhubaneswar| Lucknow|Patna|Bengaluru|Chennai|Vijayawada|Vizag |Tirupati | Kukatpally| Kolkata
: 49 : Pre GATE - 2017_EE

55. The synchronous speed of an induction motor is 900 rpm. Under a blocked rotor condition, the
input power to the motor is 45 kW at 193.6A. The stator resistance per phase is 0.2  and the
transformation ratio is a = 2. The ohmic value of the rotor resistance per phase is
(A) 0.05 (B) 0.1 (C) 0.25  (D) 0.8 
Ans: (A)

Sol: PBR  3I 2BR R 1  R 12 
 3I 2BR R 1  a 2R 2 
45  103
0.2  4  R 2 
3  193.62
R2 = 0.05
DISTRACTOR LOGIC
Option:B 
PBR  3I 2BR R 1  R 12 
 3I 2BR R 1  aR 2 
45  103
0.2  2  R 2 
3  193.62
R2 = 0.1
Option:C 
PBR  I BR R 1  R 12
2

 I 2BR R 1  a 2R 2 
45  103
0.2  4  R 2 
193.62
R2 = 0.25 
Option:D 
PBR  3I 2BR R 1  R 12 
 R 
 3I 2BR  R 1  22 
 a 
R2 45  103
0.2  
4 3  193.62
R2 = 0.8 

ACE Engineering Academy Hyderabad|Delhi|Bhopal|Pune|Bhubaneswar| Lucknow|Patna|Bengaluru|Chennai|Vijayawada|Vizag |Tirupati | Kukatpally| Kolkata


: 50 : Pre GATE - 2017_EE

56. A protection system is installed with 1 12 cycle circuit breaker. If the relay issued TRIP signal to
breaker at voltage zero instant, the breaker operating time is
(A) 30ms (B) 35ms (C) 25 ms (D) 40ms

Ans (B)
Sol: Voltage zero means current is peak
For 50Hz,
1
Time period (T) = m sec  20m sec
50

5ms

20ms

1 12 cycle circuit breaker circuit breaker opening time = 20 +10 =30ms


Arcing time = 5ms
Circuit breaker operating time
= opening time + arcing time
= 30+5 = 35ms
DISTRACTOR LOGIC
Option-A: Voltage zero means current is peak for 50Hz,
1
Time period (T) = m sec  20m sec
50

20ms

1 12 cycle circuit breaker circuit breaker opening time = 20+10 =30ms

ACE Engineering Academy Hyderabad|Delhi|Bhopal|Pune|Bhubaneswar| Lucknow|Patna|Bengaluru|Chennai|Vijayawada|Vizag |Tirupati | Kukatpally| Kolkata


: 51 : Pre GATE - 2017_EE

Option-C: Voltage zero means current is peak


For 50Hz,
1
Time period (T) = m sec  20m sec 5ms
50
20ms

1 12 cycle circuit breaker circuit breaker opening time = 20 +10 =30ms


Arcing time = 5ms
Circuit breaker operating time
= opening time – arcing time
= 30 – 5 = 25ms
Option-D: Voltage zero means current is peak
For 50Hz,
1
Time period (T) = m sec  20m sec
50 10ms
20ms

1 12 Cycle circuit breaker circuit breaker opening time = 20 +10 =30ms


Arcing time = 10 ms
Circuit breaker operating time
= opening time – arcing time
= 30 + 10 = 40ms

57. The input signal x(t) = 4+cos(4t) – sin(8t) is passed though a filter with impulse response
h(t) = Sinc2(t) cos(16t). Then the output is_________

1
(A) cos(4t ) (B) –0.5sin(8t) (C) 0 (D) 4+cos(4t)–sin(8t)
4

ACE Engineering Academy Hyderabad|Delhi|Bhopal|Pune|Bhubaneswar| Lucknow|Patna|Bengaluru|Chennai|Vijayawada|Vizag |Tirupati | Kukatpally| Kolkata


: 52 : Pre GATE - 2017_EE

Ans: (C)
H(f)
Sol: The input frequencies are f1 = 0, f2 = 2Hz, f3 = 4Hz
Sinc2(t)  Tri(f)
1/2
Tri(f  8)  Tri(f  8)
H (f ) 
2
–9 –8 –7 7 8 9 f
So no input frequencies are passed through the filter

Distractor Logic:
Option A: If you feel filter spectrum covers the frequency range from 1 – 3Hz misinterpretation of
Ans(A)
Option B: If you feel filter spectrum covers the frequency range from 3 – 5Hz misinterpretation of
Ans(B)
Option C: Correct option
Option D: If you feel that h(t) is ideal all pass filter with pass band gain of 1, that passes all frequencies.
misinterpretation of Ans(D)

58. The root loci diagram of a unity feedback system is given below. The closed loop transfer function
for K = 2, is

j 3



– j 3

2 1
(A) (B)
s  3s  3s  1
3 2
s  3s  3s  1
3 2

1 2
(C) (D)
s  3s  3s  2
3 2
s  3s  3s  3
3 2

ACE Engineering Academy Hyderabad|Delhi|Bhopal|Pune|Bhubaneswar| Lucknow|Patna|Bengaluru|Chennai|Vijayawada|Vizag |Tirupati | Kukatpally| Kolkata


: 53 : Pre GATE - 2017_EE

Ans: (D)
3
Sol: tan60 = x=1
x
j 3
Pole is at s = 1
K  60
Open loop TF G(s) =
(s  1)3
j 3
G(s)
Closed loop TF =
1  G(s)H(s)
H(s) = 1, K = 2
2
(s  1)3 2
CLTF = = 3
2 s  3s  3s  3
2
1
(s  1) 3

DISTRACTOR LOGIC:
Option A: Three poles are at s = 1
K 2 2
 TF =   3
(s  1) 3
(s  1) 3
s  3s  3s  1
2

Option B: Three poles are at s = 1


1 1
TF =  3
(s  1) 3
s  3s  3s  1
2

Option C: Three poles at s = 1


1
OLTF =
(s  1)3
1
(s  1)3 1
CLTF = = 3
1 s  3s  3s  2
2
1
(s  1)3

ACE Engineering Academy Hyderabad|Delhi|Bhopal|Pune|Bhubaneswar| Lucknow|Patna|Bengaluru|Chennai|Vijayawada|Vizag |Tirupati | Kukatpally| Kolkata


: 54 : Pre GATE - 2017_EE

59. The following counter is a

1 T0 Q0 T1 Q1
x

Q0 Q1

(A) up counter when x = 0 (B) down counter when x = 0


(C) up counter when x = 1 (D) always up counter

Ans: (A)
Sol: If x = 0  T1= Q0

1 T0 Q0 T1 Q1

Q0 Q1

Present state Next state


T1 T0
Q1 Q0 Q1 Q0
0 0 0 1 0 1
0 1 1 1 1 0
1 0 0 1 1 1
1 1 1 1 0 0

 If x = 0 given counter is up counter


If x = 1 T1 = Q0

1 T0 Q0 T1 Q1

Q0 Q1

ACE Engineering Academy Hyderabad|Delhi|Bhopal|Pune|Bhubaneswar| Lucknow|Patna|Bengaluru|Chennai|Vijayawada|Vizag |Tirupati | Kukatpally| Kolkata


: 55 : Pre GATE - 2017_EE

Present state Next state


T1 T0
Q1 Q0 Q1 Q0
0 0 1 1 1 1
0 1 0 1 0 0
1 0 1 1 0 1
1 1 0 1 1 0

 If x = 1 given counter is down counter.

60. Two identical generators are connected in parallel to a common bus. Sequence reactances of each
generator are X1 = X2 = j0.2 pu and X0 = j0.05 pu. The neutral of one of the generator is connected
to ground by reactance of j0.05 pu and other generator neutral is isolated from ground. A most
common short circuit fault is taking place at the common busbar with a fault reactance of j0.05 pu.
If the operating voltage of busbar is 13.2 kV, then the voltage of zero sequence network in kV is
(A) 2.77 (B) 3.54 (C) 6.66 (D) 4.42

Ans: (A)
j0.2
Sol: X1eq   j0.1
2
j0.2 +
X 2 eq   j0.1 Z1eq j0.1
VR1
2
X0eq = X0 +3Xn+3XF 

= 0.05+30.05+30.05 +
= 0.35 Z2eq j0.1 VR2 3Zf
0.15
E R1 
I R 0  I R1  IR0
Z1eq  Z 2eq  Z0eq +
Z0eq j0.2 VR0
1.0
 
j0.1  j0.1  j0.35
1.0
I R 0  I R1   1.82  90
j 0.55
VR 0  I R 0 X 0 eq  1.82  90  0.290
VR 0  0.364  0.364pu
13.2
VR 0  0.364   2.77kV
3

ACE Engineering Academy Hyderabad|Delhi|Bhopal|Pune|Bhubaneswar| Lucknow|Patna|Bengaluru|Chennai|Vijayawada|Vizag |Tirupati | Kukatpally| Kolkata


: 56 : Pre GATE - 2017_EE

DISTRACTOR LOGIC
Option B: X1eq  j0.2 Z1eq j0.1
+
VR1
X 2 eq  j0.2

X0eq = X0 +3Xn+3XF
+
= 0.05+30.05+30.05 Z2eq j0.1 VR2 3Zf
0.15
= 0.35 
E R1 1.0 IR0
I R 0  I R1   +
Z1eq  Z 2eq  Z0eq j0.2  j0.2  j0.35
Z0eq VR1 VR0
1.0 j0.2
I R 0  I R1   1.33  90 
j 0.75
VR 0   I R 0 X 0 eq  1.33  90  0.3590
VR 0  0.465  0.465 pu
13.2
VR 0  0.465   3.54 kV
3

j0.2 +
Option C: X1eq   j0.1 Z1eq j0.1
2
j0.2 
X 2 eq   j0.1
2
+
X0eq = X0 +3Xn + 3Xf Z2eq j0.1 VR2 3Zf
= 0.05+30.05 + 3(0.05) 0.15

= 0.35
IR0
E R1 1.0 +
I R 0  I R1  
Z1eq  Z 2eq  Z0eq j0.1  j0.1  j0.2 Z0eq j0.2 VR0

1.0 
I R 0  I R1   2.5  90
j 0.4
VR 0   I R 0 X 0 eq  2.5  90  0.3590

VR 0  0.875  0.875 pu
13.2
VR 0  0.875   6.66 kV
3

ACE Engineering Academy Hyderabad|Delhi|Bhopal|Pune|Bhubaneswar| Lucknow|Patna|Bengaluru|Chennai|Vijayawada|Vizag |Tirupati | Kukatpally| Kolkata


: 57 : Pre GATE - 2017_EE

Option D: X1eq  j0.2


+
X 2 eq  j0.2 Z1eq j0.1
VR1
j0.2
X0eq = X0 +3Xn + 3Xf

= 0.05+30.05 + 3 0.05
+
= 0.35 Z2eq j0.1 VR2 3Zf
0.15
E R1 1.0 
I R 0  I R1  
Z1eq  Z 2eq  Z 0eq j0.2  j0.2  j0.2 IR0
+
1.0 Z0eq j0.2 VR0
I R 0  I R1   1.66  90
j 0.6 
VR 0   I R 0 X 0 eq  1.66  90  0.3590

VR 0  0.581  0.581pu
13.2
VR 0  0.581  4.42 kV
3

z cos z
61.  
2
dz  ? where ‘C’ is | z – 1| = 1
z  
C

 2
(A) i (B) –i (C) i2 (D) –i2

Ans: (D)
 3.14
Sol: z    1.57 is a pole of order ‘2’ lies inside ‘C’
2 2
z cos z 
 dz  2i f 1   (where f(z) = zcosz)
2
C  2
z  
 z

= 2i  
 2 
= –2i

ACE Engineering Academy Hyderabad|Delhi|Bhopal|Pune|Bhubaneswar| Lucknow|Patna|Bengaluru|Chennai|Vijayawada|Vizag |Tirupati | Kukatpally| Kolkata


: 58 : Pre GATE - 2017_EE

62. f(x, y) = (x2 + y2 + 6x + 12) has


(A) maximum value at (–3, 0) (B) minimum value at (–3, 0)
(C) maximum value at (0, –3) (D) minimum value at (0, –3)

Ans: (B)
f
Sol:  2x  6  0 ………. (1)
x
f
 2 y  0 ……………. (2)
dy
By solving (1) & (2) for (x, y) = (–3, 0) is the stationary point
 2f  2f  2f
r  2 , s   0 , t  2
x 2 xy y 2
 At (–3, 0); (rt – s2) = 4 & r = 2
 we get minimum value of f(x ,y)

63. An ideal residential distribution transformer is connected as shown in the Fig. Assume that the two
series-connected secondary windings are identical. Determine the primary current and minimum
kVA rating of a 2400:240/120V transformer required to sustain this load without risk of winding
over-temperature.
I1
I2 +
+ 1200 V
– 20
24000 V
V +

I3 1200 V 10

Ideal

(A)1.8 A, 5.76 kVA (B) 1.03 A, 4.07 kVA


(C) 1.8 A, 2.88 kVA (D) 1.03 A, 2.03 kVA
Ans: (A)
2400
Sol: I 2   120 A
20
1200
I3   I 2  120  120  240 A
10
120 120 120
I1  I2  I3   360  1.80 A
2400 2400 2400
ACE Engineering Academy Hyderabad|Delhi|Bhopal|Pune|Bhubaneswar| Lucknow|Patna|Bengaluru|Chennai|Vijayawada|Vizag |Tirupati | Kukatpally| Kolkata
: 59 : Pre GATE - 2017_EE

Since I 3 is the larger secondary current, the rating is dictated by the lower secondary winding;
thus,
S R  2V3 I3  2 120  24   5.76 kVA
DISTRACTOR LOGIC
2400 o
Option B: I2   120 o A
20
1200
I3   I 2  120  120  16.970 A
10
120 120 120
I1  I2  I3   360  1.030 A
2400 2400 2400
S R  2V3 I3  2 120 16.97   4.072 kVA

2400 o
Option C: I 2   120 o A
20
1200
I3   I 2  120  120  240 A
10
120 120 120
I1  I2  I3   360  1.80 A
2400 2400 2400
S R  2V3 I3  120  24   2.88 kVA

2400 o
Option D: I 2   120 o A
20
1200
I3   I 2  120  120  16.970 A
10
120 120 120
I1  I2  I3   360  1.030 A
2400 2400 2400
S R  2V3 I3  120 16.97   2.036 kVA

64. If the probability that a man aged ‘X’ years will die within a year be ‘P’ then the chance that out of
5 men A, B, C, D and E each aged ‘X’ years, ‘A’ will die during the year and be the first person to
die is ___________
P1  P  1  1  P 
4 5
(A) (B) (C) 1 – (1 – P)5 (D) P(1 – P)4
5 5

ACE Engineering Academy Hyderabad|Delhi|Bhopal|Pune|Bhubaneswar| Lucknow|Patna|Bengaluru|Chennai|Vijayawada|Vizag |Tirupati | Kukatpally| Kolkata


: 60 : Pre GATE - 2017_EE

Ans: (B)
Sol: The probability that a man aged ‘X’ year will not die within a year is (1 – P)
 The chance that none of the five persons die within a year is (1 – P)5
 1 – (1 – P)5 gives atleast one of the five dies within a year
1  1  P 
5
 Anyone has the same chance of being dead first, hence the required probability  .
5
NO-DISTRACTOR LOGIC

65. An RC low pass filter has the impulse response h(t) = e–t u(t). The response of the
system due to the input x(t) = e2t u(–t) is__________
1 2t 1 1 2t 1
(A) e u ( t )  e t u ( t ) (B) e u ( t )  e t u ( t )
3 3 3 3
1 1
(C)  e 2 t u ( t )  e t u ( t ) (D) e 2 t u ( t )  e  t u ( t )
3 3
Ans: (A)
1
Sol: H(s)  ;  >–1
s 1
1
X(s)  ;<2
s2
Output ROC = ( > –1 )  ( < 2) = –1 <  < 2
1 1 3 1 3
Y(s)  X(s)H(s)   
(s  2)(s  1) s  2 s  1
Based on the output ROC, take inverse Laplace transform
1 1
y( t )  e 2 t u (  t )  e  t u ( t )
3 3
Distractor Logic:
Option A: Correct Answer
Option B: In the partial fraction if we feel pole ‘2’ is right sided & pole ‘–1’ is left sided
Option C: In the partial fraction expansion if we take negative sign of Y(s) as it is
1
Option D: In the partial fraction expansion if we miss multipliers
3

ACE Engineering Academy Hyderabad|Delhi|Bhopal|Pune|Bhubaneswar| Lucknow|Patna|Bengaluru|Chennai|Vijayawada|Vizag |Tirupati | Kukatpally| Kolkata

You might also like